Independent Condition Probability Problem

Click For Summary
The discussion revolves around calculating probabilities related to a video game scenario where a player faces four opponents, each with an 80% chance of being defeated independently. For part (a), the correct probability of defeating all four opponents is calculated as 0.4096. In part (b), the participant incorrectly calculates the probability of defeating at least two opponents, acknowledging that it requires summing the probabilities of defeating two, three, and four opponents. The participant expresses uncertainty about part (c), which involves determining the probability of defeating all four opponents at least once over three game plays, suggesting a similar approach to part (b). Clarification on these probability calculations is sought to ensure accuracy.
Giuseppe
Messages
42
Reaction score
0
Hello, I had this problem to do for statistics. I am pretty sure I did the first two parts correct. The third, I am not so sure about. I'd appreciate and guidance.

A player of a video game is confronted with a series of four opponents and an 80% probability of defeating each opponent. Assume that the results from opponents are independent (and that when the player is defeated by an opponent the game ends
(a) What is the probability that a player defeats all four opponents?
(b) What is the probability that a player defeats at least two of the opponents in a game?
(c) If the game is played three times, what is the probability that the player defeats all four opponents at least once?

My work:
(a) Since the probability of beating a player is 80 % and is also independent, then the probability of winning a game is .08 x .08 x .08 x .08
which comes out to .4096

(b) Since here the player only needs to beat two opponents so .08 x .08
which comes out to .64

(c) I am not too sure here, any help?
 
Physics news on Phys.org
(a) looks correct. (b) unfortunately is not that simple. If I remember correctly how to do this correctly, the answer is the sum of the probability of defeating two, three and four opponents.

(c) uses a similar strategy to (b).
 
Question: A clock's minute hand has length 4 and its hour hand has length 3. What is the distance between the tips at the moment when it is increasing most rapidly?(Putnam Exam Question) Answer: Making assumption that both the hands moves at constant angular velocities, the answer is ## \sqrt{7} .## But don't you think this assumption is somewhat doubtful and wrong?

Similar threads

Replies
3
Views
8K
  • · Replies 7 ·
Replies
7
Views
2K
Replies
9
Views
2K
  • · Replies 19 ·
Replies
19
Views
4K
  • · Replies 5 ·
Replies
5
Views
2K
Replies
1
Views
2K
  • · Replies 8 ·
Replies
8
Views
2K
Replies
1
Views
1K
  • · Replies 12 ·
Replies
12
Views
2K
  • · Replies 18 ·
Replies
18
Views
3K